Download as pdf or txt
Download as pdf or txt
You are on page 1of 24

For Capgemini

Boat & Stream


Formulas to remember:
Downstream/Upstream:
1. In water, the direction along the stream is called downstream. And, the direction against the stream is called upstream.
2. If the speed of a boat in still water is u km/hr and the speed of the stream is v km/hr, then:
Speed downstream = (u + v) km/hr.
Speed upstream = (u - v) km/hr.
3. If the speed in downstream is a km/hr and the speed in upstream is b km/hr, then:
Speed in still water = (a + b)/2 km/hr.
Rate of stream =(a - b)/2 km/hr
Question 1
A motorboat can cover 10 1/3 km in 1 hour in still water. And it takes twice as much as time to cover up than as to cover down the
same distance in running water. The speed of the current is:
a)3 4/9 km/hr b) 2 1/3 km/hr c) 4 km/hr d) none of these
Answer : a) 3 4/9 km/hr
Solution :
Let the speed of upstream be X km/hr.
Then, speed in downstream = 2X km/hr (since boat takes twice as much as time to cover up than as to cover down the same distance
in running water).
Speed
= 3X/2 km/hr.

in

still

water

(2X+X)/2

km/hr.

(formula

3)

Given that, boat covers 10 1/3 km in 1 hour in still water.


Therefore,
X = 62/9
So,

3X/2

speed

in

10

upstream

1/3

62/9

km/hr.

And, speed in downstream = 2 x 62/9 = 124/9 km/hr


Hence,

speed

= 62/9x2 = 34/9 = 3 4/9 km/hr.

of

the

current

[(124/9

62/9)]/2

km/hr

Question 2
A man can row a certain distance downstream in 2 hours while he takes 3 hours to come back. If the speed of the stream be 6 km/hr
then the speed of the man in still water is:
a) 15km/hr b) 30km/hr c) 25km/hr d) 29km/hr
Answer : b) 30km/hr
Solution :
Let the speed of the man in still water be X km/hr.
Given

that,

speed

of

Therefore,
speed
in
downstream
And, speed in upstream = (X-6) km/hr

the
=

stream

(X+6)

km/hr

(by

6
using

km/hr.
formula

2)

Distance covered in downstream in 2 hours = (X+6)2 km


Distance covered in upstream in 3 hours = (X-6)3 km
Therefore,
2X+12

(X+6)2

(X-6)3
3X-18

X = 30km/hr.
Question 3
A man can take the same time to row 13 km downstream and 7 km upstream. His speed in still water 5 km/hr. The speed of the
stream is:
a) 5/2 km/hr b) 3/2 km/hr c) 7/2 km/hr d) 2 km/hr
Answer : b) 3/2 km/hr
Solution :
Given
Let
Then

that,

the

the

speed

speed
speed

in

of

still

the
downstream

in

water

stream

be
=

km/hr

km/hr.
km/hr

13/(5+X)

(5+X)

And, speed in upstream = (5-X) km/hr


The
time
taken
to
The time taken to cover 7 km upstream = 7/(5-X)
Therefore,
13(5-X)
65
30
X = 30/20 = 3/2

cover

13

13/(5+X)

km

downstream

7/(5-X)

=
-

13X
=

7(5-X)
35+7X
20X

Hence the required answer is 3/2 km/hr.


Question 4
A boat takes 7 hours to cover 24 km distance and comes back. And, it can cover 2 km with the stream in the same time as 1.5 km
against the stream. The speed of the stream is:
a) 1 km/hr b) 2 km/hr c) 3 km/hr d) 4 km/hr
Answer : a) 1 km/hr
Solution :
Let
the
boat
takes
Then, speed in downstream = (2/X) km/hr

hours

to

cover

km

in

downstream.

and, speed in upstream = (1.5/X)km/hr


Given that, the boat takes 7 hours to cover 24 km distance and comes back.
That
24X/2

is,

24/(2/X)

+
48X/3

168X/6
X = 42/168 = 1/4

24/(1.5/X)

7
7

So,
speed
in
downstream
Speed in upstream = 1.5/X = 1.5 /(1/4) = 6 km/hr.

Speed

(8-6)/2

of

the

stream

2/X

km/hr

/(1/4)

(by

using

the

km/hr

formula

3)

= 1 km/hr.
Question 1
The rate of current and boat in still water is 3 km/hr and 18 km/hr respectively.The boat starts from the point A and reaches the
destination point B and returns back to A.Find the time taken by the boat for both forward and backward journey if the distance
between them is 210 km.
a) 12 hours b) 20 hours c) 24 hours d) 16 hours
Answer : c) 24 hours.
Solution :
Speed
Speed

of

the

of

the

stream
in

boat

=
still

Speed
of
the
boat
in
downstream
=
Speed of the boat in upstream = (u - v) = 18 - 3 = 15 km/hr.
Distance
Time

taken

between
by

the

boat

A
to

reach

v
water

(u

and
(forward

journey)

=
=

v)

18

B
(in

upstream)

3
=
+

=
=

distance/speed

km/hr
km/hr

3
=

21

210
=

km/hr

km.
210/15

hours.

Time

taken

by

the

boat

to

reach

(backward

journey)

(in

downstream)

210/21

hours.

Total time = 210/15 + 210/21 hours = 14 + 10 = 24 hours.


Question 2
If the speed of the stream is 6 km/hr and the speed of a man in still water is 20 km/hr, then the distance covered by the man
downstream in 15 minutes is:
a) 3.5 km b) 4.5 km c) 6.5 km d) 5.5 km
Answer : c) 6.5 km
Solution :
Time
Speed
Speed

taken

by

the

of
of

man

the
man

the

stream
in

15

minutes

=
still

Speed
of
the
man
in
downstream
=
Distance Covered = time x speed = 26 x 15/60 km = 6.5 km.

v
water

20

=
6

15/60

=
u

hour.

6
+

km/hr
km/hr.

20

26

km/hr.

Question 3
The speed of a motor-boat in still water is 12 km/hr and the speed of the current is 3 km/hr. If the boat takes 3 hours to arrive at a
place and return back, then how far is the place?
a) 17 km b) 18 km c)19 km d) 20 km
Answer : a) 17 km.
Solution :
Speed
Speed

of
of

the
motor-boat

the

stream
in

=
still

Speed
of
the
boat
in
downstream
Speed of the boat in upstream = (u - v) = 12 - 3 = 9 km/hr
Time
Let
Time
Time
Total
(3X

taken

by

the

the
taken
taken

boat
required

by
by

time

the
the
=

X
=
Hence the distance is 17 km (approximately).

to

water

(u

v)

arrive
and
distance

boat
boat

in

=
=

12

return
be

=
5X)/45

135/8

=
u

3
=
+

=
=
+
=

Average Problems
Question 1
The average of five consecutive odd integers is 113. What is the second smallest of them?

=
X

X/15
16.875

km/hr
km/hr

12

back

downstream
upstream

in

15

km/hr

hours.
km.

X/15
X/9

hr.
hr.
X/9
3
km.

a) 111 b) 115 c) 109 d) none of these


Answer : a) 111
Solution :
Let the five consecutive odd numbers be X, X+2, X+4, X+6 and X+8
Given that, their average = 113.
That

is,

(X

(5X
5(X+4)

X+2

X+4

20)

X+6

/
5

X+8)

113
113
113

X = 113 - 4 = 109.
Second smallest number is X + 2 = 109+2 = 111.
Question 2
The average of 6 consecutive even numbers is 207. What will be the sum of the smallest and largest number?
a) 210 b) 408 c) 414 d) 208
Answer : c) 414
Solution :
Let the six consecutive odd numbers be X, X+2, X+4, X+6, X+8 and X+10
Given that, their average = 207
That

is,

(X

(6X+30)
6(X+5)

X+2
/
/

X+5
X = 202.

X+4

X+6

6
6

X+8

X+10)/6

=
=

Therefore the smallest number = 202 and the largest number = x+10 = 202+10 = 212
Required sum = 202+212 = 414.
Question 3
The average of the four consecutive even integers is 3645 less than their sum. What is the last of these numbers?
a) 1218 b) 2146 c) 3212 d) none of these
Answer : a) 1218
Solution :

207
207
207
207

Let the 4 consecutive even integers be X, X+2, X+4 and X+6.


Then,
their
average
=
(X
+
X+2
And, their sum = (X + X+2 + X+4 + X+6) = 4X+12...(2)

X+4

X+6)/4

(4X+12)/4

(X+3)

....(1)

Given that, average is 3645 less than the sum.


That

is,

from

(1)

and

(2),

3X
X = 1212

4x+12

3645

X+3

3636

Therefore, required last number = X+6 = 1212 + 6 = 1218.


Question 4
If the average of seven consecutive odd numbers is 2117 then what will be the average of first four?
a) 2111 b) 2112 c) 2113 d) 2114
Answer : d) 2114
Solution :
Let the 7 consecutive odd numbers be X, X+2, X+4, X+6, X+8, X+10 and X+12.
We have to find the average of X, X+2, X+4 and X+6
That is, (X + X+2 + X+4 + X+6) / 4 = (4X+12) / 4 = X+3 ...(1)
Given that, the average that 7 numbers = 2117
That

is,

(7X
(X

(X

X+2

X+4

X+6

42)
+

+
/

6)

X+8

X+10

X+12)/7
=

2117
2117
2117

X = 2111
Putting the above X value in (1), we get, X+3 = 2111+3 = 2114
Hence the required answer is 2114.
Question 1
The average weight of 50 men was 72.The weight of the 46th person was 68 which has been wrongly entered as 46.Find the
corrected average weight.
a) 73 b) 78 c) 75 d) none of these.
Answer : d) none of these.
Solution :

Average

weight

of

50

men

72.

Sum of weights of these 50 men = 72 x 50.


Since

the

weight

68

is

wrongly

entered

as

46.

Amount of weight decreased in the sum = 68 - 46 = 22.


Therefore,
And,
required

the
average

corrected
of
50

men

new
=

(72

sum
x
50

=
+

22)

72
/

50

x
=

50
3622

+
/

50

22.
72.44

Hence the answer is option d.


Question 2
In an entrance test, a candidate's point were wrongly taken as 42 instead of 24. Because of that, the average score for the exam got
increased
by
of candidates who have attended the exam.

3/4.

Find

the

number

a) 24 b) 12 c) 36 d) 48
Answer : a) 24.
Solution :
Let
there
be
And, let X be incorrect average score.
Incorrect

total

candidates

marks

Since,
the
points
2.4
Corrected total marks = (X x N) - (42 - 24) = NX - 18 ....(1)
Since
the
average
increased
by
Correct average = (X - 3/4)N = NX - 3N/4 ....(2)
Equating
NX
3N

(1)

3/4,

is

then

18

the

correct

18

average

NX

4.2.

3/4.

get,
-

x
4/3

N
as

we

18

exam.

x
entered

(2),

N
=
Hence, the number of candidates is 24.

the

X
wrongly

and

in

3N/4
4
24.

Question 3
The average of 30 non-negative numbers is 30. Out of these numbers, the average of 10 numbers is 28 and that of the other 18
numbers
is
32.
The
average
of
the
remaining numbers is:
a) 44 b) 22 c) 11 d) none of these.
Answer : b) 22.
Solution :

Average

of

30

numbers

30.

10

numbers

28

18

numbers

32

Sum of 30 numbers = 30 x 30 = 900


Average

of

Sum of theses 10 numbers = 28 x 10 = 280.


Average
of
Sum of these 18 numbers = 18 x 32 = 576.
Now,
We
Sum

number

of

have
to
of
these

remaining

numbers

find
remaining

the
numbers

30

average
=
900

of
-

10

18

these
(280
+

two
576)

2.

numbers.
=
44.

Average of remaining numbers = 44/2 = 22.

Mixture Problems
Alligation

It is the rule that enables us to find the ratio in which two or more ingredients at the given price must be mixed to produce a mixture of
desired price.
Alligation

If
two
Quantity of cheaper /

ingredients
quantity of dearer = C.P of

dearer

are
mean price /

mixed,
mean price

then
of cheaper.

C.P

We present as under:

Therefore, (Cheaper quantity) : (Dearer quantity) = (d - m) : (m - c).


Question 1
Two cans A and B contains milk worth Rs.7 per litre and Rs.9 per litre respectively. If the contents of A and B are transferred to
another can C in the ratio 3 : 7 then the cost per litre of the mixture in can C is:
a) Rs.9.40 b) Rs.10.10 c) Rs.9.40 d) Rs.8.40
Answer : d) Rs.8.40
Solution :
Cost
Cost

of
of

1
1

litre
litre

of
of

Let
the
Applying the rule of alligation,

Therefore,
Given

(Cheaper
ratio

quantity)
=

A
B

=
=

Rs.7
Rs.9

mean

:
3

(Dearer
/

=
=

cheaper
dearer

price

quantity)
7

(d
=

m)
9

quantity.
quantity.

be

(m
-

c)
X

(9
/

Rs.X.

X)
X

(X
-

7)
7

63

7X

X
Hence, the required mean price = Rs.8.40.

3X

21

8.4.

Question 2
Two qualities of rice at Rs.63 per kg and Rs.67.50 per kg are mixed with another quality of rice in the ratio 2:2:3. The final mixture sold
at Rs.76.50 per kg then the rate of third quality rice per kg was:
a) Rs.87.50 b) Rs.91.50 c) Rs.81.50 d) Rs.99.50
Answer : b) Rs.91.50
Solution :
Given
Their

that,
average

first

and
price

2nd

varieties
Rs.(63

are

mixed
67.50)

in

equal
2

proportion.(2:2)
Rs.65.25

The new mixture is formed by mixing two varieties, one at Rs.65.25 per kg and the other at Rs.X per kg in the ratio 4:3. (note that,
2:2:3
becomes
4:3).
Given
We

that,

mean
have

price

of
to

new

mixture
find

Rs.76.50
X.

Applying the rule of alligation,

Therefore,

(Cheaper

X
3X

quantity)

(Dearer

76.50

quantity)

(d

/
229.50

X
Hence, the required price of 3rd quality rice = Rs.91.50.

m)

11.25

(m
=

c)

=
4/3
45
91.50

Question 3
In what ratio, a liquid A of cost Rs.31 per litre should be mixed with liquid B of cost Rs.36 per litre, so that the cost of the liquid of
mixture is Rs.32.35 per litre?
a) 2:1 b) 3:1 c) 3:2 d) 4:3
Answer : b) 3:1
Solution :
Cost
Cost

of
of

1
1

litre
litre

Given,
mean
Applying the rule of alligation,

of
of

liquid
liquid

A
B
price

=
=

Rs.31
Rs.36

=
=
=

cost
cost

of
of
Rs.

cheaper
dearer

quantity.
quantity.
32.25

Therefore,

required

ratio

(Cheaper

quantity)

=
3.75
Hence, the answer is option b.

(Dearer

quantity)

(d

1.25

m)

(m

c)
3:1.

Area Problems

Question 1
The dimension of a rectangular shaped wall was X x Y. If the breadth is increased by 40% and the length is decreased by 30% then
the % of area of the new wall compared with previous one is:
a) 88% b) 98% c)68% d)78%
Answer : b) 98%
Solution :
Dimensions
Without
loss

of

the

of

generality,

wall
that,

assume

=
is

X
length

x
and

Y.
breadth.

is

Area of original wall = XY unit2.


New

length

(after

decreasing

New
breadth
(after
increasing
New area = 70X/100 x 140Y/100 = 49XY/50.
Therefore,

required

(new

area

30%)
40%)

original

area

100

30X/100
+

40Y/100

((49XY/50)

XY)

70X/100

140Y/100.

100=

98%.

Hence, the answer is 98%.


Question 2
If the area of a square shaped field decreases by 36%, then the % of each side of the field decreases by:
a) 15% b) 12% c) 20% d) 19%
Answer : c) 20%
Solution :
Let
Then,

us

assume
its

that
original

the

%
decrease
New area = 100 - 36 = 64 unit2.
Therefore,

new

i.r.,
decrease
Decreasing
%
=

Hence, the answer is 20%.


Question 3

area
=

in

side
on
(units

original
sides

of
of

the

10
units
decreasing
/

of
the
sqrt(100)

field
=

area

field
=
original

10

36%

sqrt(64)

units.

8)
=

=
x

2
100/10

units.
20%

unit2.
units.

100

=
(10
units)

is

100

The dimension of a black board is L m x B m. If l and b decreases by 20% and 40% respectively, then the area of the board before
alteration exceeds the area of new one by:
a) 48% b) 52% c) 39% d) 62%
Answer : b) 52%
Solution :
Let
length
Original area = (LB) m2.
New

length

(after

decreasing

20%)

(100

and

20)%

of

breadth

L=

80%

of

80L

/100

m.

4L

m.

New breadth (after decreasing 40%) = (100 - 40)% of B = 60% of B = 60B / 100 = 3B / 5 m.
New
Difference

area
in
area

=
=

original

4L/5
area

x
new

3B/5
=
Lb

area

=
-

12(LB)/25
12(LB)/25
=
13(LB)/25

m2.
m2.

Therefore, required % = difference / original area x 100 = [(13(LB)/25) / (LB) ] x 100 = 13 x 4 = 52%.

Arithmetic calculations
Question 1
A fruit seller has 16 apples and cost of each apple is Rs.12. Three persons A,B and C decided to buy that apples. If A and B gives
Rs.84 and Rs.48 to the seller for some apples, then how many apples C can buy?
a) 8 b) 11 c) 5 d) 6
Answer : c) 5
Solution :
Given

that

Then
A
Then
= Rs.60.
TC
Since

the

total
and

seller
cost

of
B

the

each

has

cost

apple

of

can
costs

16
16
buys

apples
apples

he
=

buy
then

each
x

can

apple
=

for
60/12

Rs.12
Rs.192
Rs.48.

and
Rs.(192

buy

at

12

Rs.84

apples

sells
16

for

remaining

Rs.12,

and

(84+48))

Rs.60
apples.

Hence the answer is 5.


Question 2
Thomas bought X number of sports goods for Rs.9000. If each item was cheaper by Rs.30 then with the same amount he could have
bought 50 more items than X. Find the number of items bought by Thomas ?
a) 100 b) 150 c) 75 d) 125
Answer : a) 100

Solution :
Transaction I
Thomas
have
bought
Then the cost of each item = Rs.9000/X ...(1)

items

for

Rs.9000.

Transaction II
If
each
item
was
cheaper
by
Rs.30,
he
could
Therefore, with Rs.30 discount, the amount of each item = Rs.9000 / X + 50 ...(2)

have

bought

50

more

items.

We know that the cost of each item in transaction II will be lesser than that of transaction I by Rs. 30.
i.e (1) - (2) = 30
Since he
9000/X

bought

1/X
50

50
-

more

cost

1/X
X(X

X(X
X^2

of

each
+

item

+
+

+
+

X^2
(X+150)(X-100)
X
X

items
when
9000/X

is

less
50

50X

Rs.30

then

we

50
50)

=
=
=

15000
15000

15000

0
0

=
=

cannot

-150
be

or
negative

X
Therefore,

value.

have
30

30/9000
1/300

50)
50X

by

=
X

100
100.

Hence the answer is 100.


Question 3
On Republic day, chocolates were to be distributed among 350 kids in a play school. But 140 kids were absent on that day and each
kid present got 3 chocolates more. Find the total number of chocolates bought for the distribution.
a) 3200 b) 2750 c) 2950 d) 1575
Answer : d) 1575
Solution :
Let

the

total

number

of

Total
number
of
kids
in
the
school(when
If everyone is present, number of chocolates per kid = X/350 chocolates ...(1)
But,

140

i.e (2)-(1)=3

everyone

kids

Therefore,
total
number
of
kids
present
Therefore, number of chocolates per present kid = X/210 chocolates ...(2)
210 kids were present and each kid gets 3 extra.

chocolates

be

is

present)

X.
=

are
=

350

350.

absent.
-

140

210

Or X/210 - X/350=3
350X - 210X / (210 x 350)=3
140X / 210 x 350=3
X = 3 x 210 x 350 / 140 = 1575
Then the required answer is 1575.
Question 4
8 friends planned to go to hotel for dinner and to share the bill amount equally. If one of them have forget to bring the wallet, then what
will be the extra amount contribute by each to pay the bill of Rs.1904 ?
a) Rs.28 b) Rs.54 c) Rs.34 d) Rs.38
Answer : c) Rs.34
Solution :
Given
Actual
If
one
of
Then
the

that
share
8
extra

the

bill

of
is
left,
amount

each
then
given

amount
Rs.1904/8

the
by

sharing
each

amount
=
=
Rs.(272

Rs.1904
Rs.238

=
Rs.1904/7
238)

=
=

Rs.272
Rs.34.

Hence the answer is Rs.34

Look And Say


Note :
A look-and-say sequence is a sequence of integers, expressed in decimal notation, where each successive term is generated by
describing the previous one.
For instance, if x1 (the first term of the sequence) is 1, the next term is the description of this term, 11 ("one 1"), which is described by
21 ("two 1's"), which is described by 1211 ("one 2 one 1"), etc.; the series continues 111221, 312211, 13112221, ...
Question 1
Here is a sequence of numbers: 64 1614 1116114 31163114 132116132114. It seems to be strange sequence, but yet there is a
system behind it. What is the number of digits in the next term in this sequence?
a) 20 b) 12 c) 16 d) 18
Answer : a)20
Solution :
Given
sequence
It is a look-and-say sequence.

is

64

1614

1116114

31163114

132116132114.

The

first

The
2nd

next
term

term
is
is
described

Then
And

the
4th
term
the
5th
term
is

term

is

the
description
of
the
first
in
the
3rd
term
as
11161114,
is
31163114
(
132116132114
(one
3

three
two

1
1

term,
(one

64.

1614
1
one

one
one
6

6
one

(one
6
6
one
1
three
1
3
two

one
one

4).
4).

one
1
one

4).
4)

Then the required term is 11131221161113122114 (one 1 one 3 one 2 two 1 one 6 one 1 one 3 one 2 two 1 one 4).
Therefore the number of digits in the required term is 20.
Question 2
If 9 is the 1st term of a look-and-say sequence then what will be its 6th term?
a) 1113122119 b) 1113222119 c) 1113122119 d) 1113421119
Answer : c)1113122119.
Solution :
The

1st

Then
2nd

term

the

is

successive
term

3rd
4th

are:
19

term
term

5th
Then

9.

terms
=
=

term

1119
3119

the

6th

term

132119
1113122119.

Hence the answer is 1113122119.


Question 3
What will be the sum of digits in 5th term of a look-and-say sequence generated by 81?
a) 20 b) 18 c) 28 d) 24
Answer : d)24
Solution:
The
1st

look-and-say
term

2nd
3rd

term
term

4th
5th

of
of
of

term
term

Then the required sum


Hence the answer is 24.

for
sequence

the
the

of
of

Profit & Loss Problems


Question 1

sequence
the

sequence
sequence

the
the
3

81
=
=

1811.
111821.

sequence
sequence
+

is,
81.

31181211.
132118111221.

=
1

24.

Find the profit % of a sales man, if he had sold the emergency lamp at a marked price. Note that, the cost price of the emergency
lamp is Rs.640 and he can make a profit of 30% even after reducing the marked price of the emergency lamp by Rs.64.
a) 32% b) 40% c)12% d) 42%
Answer : b) 42%
Solution :
Cost
Profit

price

(C.P)
made

of

an

emergency
him

by

i.e.,
if
the
C.P
is
100
then
Here, the S.P of emergency lamp = 130/100 x 640 = Rs.13 x 64 = Rs.832
After

reducing

the

marked

price

Then
it's
marked
price
Now marked price = Rs.896 and C.P = Rs.640
If

he

sells

at

marked

Then
required
Hence the answer is 40%.

by

Rs.64,

then

Rs.640
30%

price(S.P)

sold

the

profit

=
=

selling

he

Rs.832

price

profit

it's

lamp

64

Rs.896

256/640

130.

lamp

for

Rs.832.

Rs.

896

Rs.256

Rs.640
100

40%

Question 2
A man bought a bike at the marked price of Rs.65,000 with a discount of 3% on its marked price. If the salesman makes 30% of profit
on this sale then what will be the cost price of the bike?
a) Rs.48,500 b) Rs.39,300 c) Rs.52,400 d) Rs.51,100
Answer : a) Rs.48,500
Solution :
Marked
price
Discount on marked price = 3%

of

the

Then
the
selling
price
(S.P)
ie., S.P = 97/100 x 65000 = Rs.(97 x 650) = Rs.63050
The
i.e.,
Therefore

sales
if
the

the
C.P

man
of

S.P
the

of

bike

the

made
is
bike

Rs.130
Rs.
100/130

bike

a
then
x

profit
its
63050

Rs.65000

97%

of

of
C.P
=
Rs.(100x485)
=

Rs.65000

30%.
Rs.100
Rs.48500

Hence the required answer is Rs.48500.


Question 3
If a salesman made a profit of 12% by selling an item whose marked price is 20% more than it's cost price then what will be the
discount % of the item?
a) 12/5% b) 13/3% c) 21/4% d) 20/3%

Answer : d)20/3%
Solution :
Let
the
C.P
of
Marked price is 20% more than it's cost price = Rs.120
Salesman

made

the

item

be

profit

Rs.100

of

12%.

selling

price

Then its selling price = (CP + Profit) = Rs.112


Discount

Rs.120
Required

marked

price

discount

Rs.112
8/120

=
x

100

Rs.8
%

20/3

Hence the answer is 20/3%.


Question 4
A seller gives 6% discount on the labelled price and gives 2 items free for buying every 11 items and gains 20%. Then the labelled
price is above the cost price by(in percentage):
a) 25% b) 12.5% c) 45% d) 18%
Answer : a) 25%
Solution :
The

seller

earns

gain

of

20%

Let
the
C.P
Then C.P of 13 items = 13 x Rs.100 = Rs.1300
Since

20%

profit

is

earned,

by

selling

of

S.P

13

items.

each

of

13

items

(11

items

item

Rs.1300

items

be

free)
Rs.100

120/100

Rs.1560.

And S.P of each item = Rs.1560/13 = Rs.120


Given
that,
the
seller
gives
i.e., S.P is Rs.94 then labelled price = Rs.100
And
if
Therefore,

S.P
is
Rs.1560
labelled
price
above

then
cost

discount

labelled
price
=

of

6%

price
=
Rs.
labelled
price
-

on

(100/96)
C.P
=

the

labelled

x
120
125
-

=
100

price.

Rs.125
=
25

Hence the required % is 25%.

Ratio Problems
Question 1
Ratio of the score of two teams P and Q is 5:8. If the score of team P is increased by 60% and those of Q decrease by 35%, then the
new ratio of their score becomes 9:8.Find the score of team P.
a)35 b)12 c)42 d)data inadequate
Answer : d) data inadequate

Solution :
Let
New

the

160
New

original
score

of

score
P

of

/
/

100
:

8X

40X
This

after

and
Q
increasing

after

x
decreasing

100

score

65
Therefore

of

8X

respectively.
of
5X

160%

5X

=
65%

8X
8X

of

26X

=
give

cannot

and
=

26X

5X

35%

8X

26X

be
60%

/
:

5
(given)

8
value

Hence the given data is inadequate.


Question 2
805 ml of mixture1 is mixed with 700ml of mixture2. If mixture1 has acid1 and acid2 in the ratio 4:3 and mixture2 has acid2 and acid3
in the ratio 2:5 then the amount of acid2 in the new mixture is:
a)125 ml b)545 ml c)345 ml c)625 ml
Answer : b)545 ml
Solution :
Acid1
Quantity
And
Quantity
Therefore

and
of
of

acid2

acid2
Acid2
acid2
quantity

in
in
of

is

805ml
and

of

700ml
of
acid2

in

the

mixture1
acid3

mixture2
=
in
new

ratio

805

3
the

in
700
x
mixture

2
=

7
345

4:3,
/

7
100

then

=
ratio

x
200

345
2

ml
2:5.

200ml
545ml.

Hence the required answer is 545 ml.


Question 3
A chemical solution of 2430 ml is poured into 3 bottles namely P,Q and R such that 5 ml,10 ml and 15 ml are taken out and the
remaining solution in P, Q and R will be in the ratio 3 : 4 : 5. Find the initial amount of solution in Q.
a)810 ml b)270 ml c)970 ml d)720 ml
Answer : a) 810 ml
Solution :
Total

amount

After taking 5
=
Therefore
Amount

ml,10

2400
of

of

ml and 15 ml

ml
solution

out,

of
solution
present
in

solution

is
Q

Initial amount of solution in bottle Q = New


i.e., 800 + 10 = 810 ml contained in bottle Q initially.
Question 4

2430

the remaining amount of Solution =


2400
in
now
amount

the
=
of

ratio
4/12

solution

in

=
x
Q

2430 -

(5

3
2400

Solution

=
taken

ml
10 +

15)

ml
ml

5
ml

800
out

from

The ratio of length of sides of a machine is 1/6:1/4:1/3 and the sum of length of sides is 104,. If we increase each side by 3 m then the
length of largest side will be:
a)45 m b)46 m c)49 m d)50 m
Answer : c)49 m
Solution :
The
Let

sides
the

Given
i.e.,

of

machine
sides

that
4X

are
of

the
+

in

the

ratio
machine

the

sum
6X

of

1/6

:
be

the

1/4

:
4X,6X

sides

1/3

8X

and

4:6:8
8X

104
104

m
m

X = 104/18 = 52/9
Then
The

the

Now
=

the

largest
sides

side

8X

are

length
of
49.22

the

largest
=

8
increased

side

416

52/9
by
/

=
+
m

49

416
3

9
m.

443
/
9
(approximately)

Hence the answer is 49 m

Age Problems

Question 1
The sum of the 1/2 of Sidharth's age two years from now and 1/3 of his age three years ago is twenty years, then how old is he now?
a)20years b)2years c)24years 4)4years
Answer : c)24years
Solution :
This

problem

refers,

Sidharth's

age

two

years

in

the

Let
Sidharth's
age
His age after two years from now is S + 2 and three years ago is S - 3.
From
1/2

of

age

two

years

from

future

and

three

years

now

the
now

is

two
S/3

(1/2)(S

in

be

2)

the

past.
S.

S/2

problem,
+
1

twenty,
20

1/3 of age three years ago is (1/3)(S - 3) = S/3 - 1


The
i.e.,
S/2
3S
S = 120/5 = 24

sum
S/2

of
+
+
+

these
1

S/3
2S

numbers
-

is
1
=
=

20
120

Hence, Sidhaeth is 24 years old now.


Question 2
Jo is 3 years elder than Rahul. Naren was 28 years of age when Mahi was born while Sheela was 26 years of age when Rahul was
born. If Mahi was 4 years of age when Jo was born, then what was the age of Naren and Sheela respectively when Jo was born ?
a)32 & 23years b)42 & 24years c)23 & 28years d)24 & 32years
Answer : a)32 & 23years
Solution :
Given
that,
If
Mahi
was

"
4

Naren
years

was
of

28
age

then

years
of
age
Naren's
age
is

when
28
+

Also
given
that,
If
mahi
was
4
years
of
age
From the above two statements we would have Naren's age when Jo was born is 32 years.

mahi
4

when

Jo

was
32

born"
years.

was

born.

Also given that, Jo is 3 years elder than Rahul and sheela was 26 years of age when Rahul was born.
Sheela's age when Jo was born = 26 - 3 = 23 years
Hence, the answer is 32 & 23 years
Question 3
My father was 28 years older than my brother and my mother was 23 years elder than my sister when I was born. If my brother is 10
years older than me and my mother is 4 years younger than my father, how old was my sister when I was born?
a)10yrs b)11yrs c)12yrs d)13yrs
Answer : b)11yrs
Solution :
When
My

I
father's

was
age

My Mother's age is 38
Hence the answer is 11 years.

born,
is
4

34

my
10
years

old

brother
+
and

was
28

my

sister's

10
38

=
age

is

34

year
yrs.
23

11

old,
old
year

old

Problems On Probability
Question 1
If a number selected at random from a set which contains only 2 digit numbers, find the probability that the selected number will be a
multiple of '8'.
a)1/10 b)4/45 c)1/15 d)none of these
Answer : b)4/45
Solution :

There

are

total

of

90

two

digit

numbers.

Therefore,
the
set
contains
90
numbers.
We know that the first two digit number that is a multiple of 8 is 16 and every 8th number from 17 will be a multiple of 8. (16 is 7th
number
of
the
set)
Therefore, there are 11 (integral value of 90/8 = 11) of those numbers that are divisible by '8'.(alternatively the two digit multiples of 8
are
16,24,32,40,48,56,64,72,80,88
Then, the required probability is 8/90 = 4/45

and

96)

Question 2
What is the probability that a number selected at random from the set of 3 digit numbers will be a multiple of '9' and a divisor of 900?
a)1/450 b)5/36 c)1/180 d)7/900
Answer : c)1/180
Solution :
The

three

digit

numbers

are

100,101,102,...,999

There
are
a
total
of
900
three
digit
numbers.
We know that the number 99 preceded by 100 is divisible by 9 and then every nine-th number from 101 will be divisible by '9'.
Therefore,
Now,
the

there
are
(900/9)
3
digit
divisors
of

100
of
those
numbers
900
are
900/1,
900/2,
900/3,

i.e.,
the
3
digit
divisors
of
900
are
Here, except 150 and 100 other numbers are multiple of 9.
The
remaining
5
numbers
Hence, the required ratio = 5/900 = 1/180.

are

both

900,

450,

multiple

of

that
900/4,

300,

are
900/5,

225,

180,

and

divisible
900/6

by
'9'.
and
900/9.

150

and

100.

of

900.

divisor

Question 3
What is the probability that a two digit number selected at random will be a divisor of '96' and not a divisor of 24?
a)2/45 b)1/30 c)1/45 d)1/15
Answer : a)2/45
Solution :
There
The
The
Here,
Therefore,

are
divisors
two-digit
the

two
16,32,48

totally
of
divisors

90
96
of

digit

divisors
and

two

are
96
of
96

digit

1,2,3,4,6,8,12,16,24,32,48
are
12,16,24,32,48
24
are

are

12
the

and
required

numbers.
and
and
24

96.
96.
only.
numbers.

Hence the required probability = 4/90 = 2/45


Question 4
Find the probability that a number selected at random from the set of two digit numbers will not be a multiple of 12 and not a multiple
of 4?
a)29/90 b)7/90 c)61/90 d)34/45

Answer : d)34/45
Solution :
The
There

two

digit
are

numbers
two

90

are
digit

10,11,12,...,99
numbers.

Since the set of multiples of 4 contains multiple of 12 then it's enough to find the number of multiples of 4.
The first two digit number that is divisible by 4 is 12 and every 4th number from 13 is multiple of 4.
Here,
Then

22
the

(integral
number
of

value
of
90/4)
numbers
two-digit
non-multiple
of
12

of
and

multiples
of
non-multiple
of

4
4

&
is

12
90-22

together.
=
68.

Hence the required ratio is 68/90 = 34/45.

Simplification
Question
1
Every day, a man spends 1/4 of his time to work, 3/8 of his time to sleep and the rest of the time in family. Then, during a week how
many
hours
a) 48 b) 54 c) 12 d) 63

did

he

spend

with

his

family?

Answer : d) 63
Solution :
Time
Then

taken
the

to

work
remaining

and

sleep
part

1/4
1

+
-

3/8

5/8
=

5/8

part
3/8

Remaining part of job 3/8 spend with his family.


1
Then
And,

part
3/8
for

of
a

part
of
week
he

day
a

day
spends

means
takes
(9
x

he

=
7)

takes

24/8
hours

63

24
=
with

hour
9
his

hours
family.

Hence the answer is 63 hours.


Question 2
Assume that 1/6, 1/10 and 1/3 part of a human body's weight is made of bones, skins and muscles respectively. Also assume that the
remaining weight is due to water and fluids. If a man's weight is 60 kg find the weight of water and fluids in his body.
a)1 2 kg b) 20 kg c) 24 kg d) 36 kg
Answer : c) 24 kg
Solution:
Given
1/10
Then

that,
part
made
the
remaining
part

1/6
of
(i.e.,

part

skin
weight
of

and
water

made
1/3
and

fluids)

part
=

of
1

made
(1/6

bones,
of
+

1/10

muscles.
+
1/3)

1 - (10 + 6 + 20)/60 = 1 - 36/60 = 2/5


Given

that

Then
water
and
Hence the required answer is 24 kg.

the
fluid

man's
weight

weight
=

is
x

60/5

60
=

kg.
24

kg.

Question 3
A boy spends 2/5 of his study time for maths, 3/10 of his study time for science and 1/8 of his study time for arts. If he spends
remaining 30 minutes for playing with friends then find the total time taken by him to learn science and arts.
a) 1 3/14 hours b) 2 1/7 hours c) 3 4/9 hours d) 1 3/10 hours
Answer : a) 1 3/14 hours.
Solution :
Part

of

the

time

left

(remaining

time)

(2/5

3/10

1/8)

1
33/40
=
7/40
Let the total time taken by him to study all subjects be X hours. We have just found that he had 7/40 remaining time after studying
maths,
science
and
From the question, we can say that he spends this remaining 7/40 of his time for playing and this time equals 30 minutes.
Then

7/40

of

1/2

7X
X = 20/7 hours.
Time

taken

Time
Time

taken
required

hour

(we

have

expressed

to

learn

30

minutes

as

half

40

science

3/10

to
learn
arts
=
1/8
to learn science and arts =

i.e

1/2

hour)

of

of
(1) +

hour

arts.

X
(2)

3/10

=
1/8
= 6/7 +

x
5/14

1/2

20/7

6/7

hours.

...(1)

20/7
=
5/14
= 17/14 hours =

hours.
1 3/14

...(2)
hours.

Hence the answer is 1 3/14 hours

Consecutive Numbers
Question 1
If

the

average

of

consecutive

odd

numbers

is

92,

then

the

least

number

of

those

numbers

is:

a) 89 b) 71 c) 81 d) 91
Answer : a) 89
Solution :
Let the four consecutive odd numbers be x, x+2, x+4 and x+6
We have to find the least number i.e., x
Given
4x

that

4x
4x

the

average
+

92.

Hence the required number is 89.

That

is,

x+2

12

x = 356/4 = 89

Question 2

is

368

12
-

x+4

x+6

=
=
12

92
92(4)
368
356

If the sum of five consecutive even numbers is 1580, then find the average of the next five consecutive even numbers
a) 326 b) 312 c) 325 d) 318
Answer : b) 312
Solution:
Let the five consecutive even numbers be x, x+2, x+4, x+6 and x+8
The
i.e.,

sum
x

of
+

5x
5x

x+2

the
+

above
x+4

+
=

five
x+6

numbers
+
x+8

20
1580

is
=

1580.
1580
1580
20

x = 312
Therefore,
the
5
even
Or 312, 314, 316, 318 and 320

numbers

are

312,

312+2,

312+4,

312+6

and

312+8

Then the next 5 consecutive even numbers are 322, 324, 326, 328 and 330
Now, the required average = 322+324+326+328+330 / 5 = 1630/5 = 326.
Hence the answer is 326.
Question 3
If the sum of the 4 consecutive even numbers is 4 more than three times the largest number, then the average of those numbers is:
a) 32 b) 12 c) 25 d) 13
Answer : d) 13
Solution :
Even numbers can be represented to have the form 2n where n = 0,1,2,3....
Based on our above understanding, any four consecutive even numbers can be represented as,
2k, 2k + 2, 2k + 4, 2k + 6
Now, adding these we get,
2k + (2k + 2) + (2k + 4) + (2k + 6) = 8k + 12
But the above sum is 4 more than 3 times the largest number(which is 2k + 6) .
i.e., 8k + 12 = 4 + 6k + 18
2k

k=5
Therefore, the sum of the numbers = 8k + 12 = 52 (by substituting the value of k)

10

Then the required average = 52/4 = 13

You might also like